« first day (2332 days earlier)      last day (2694 days later) » 

4:00 PM
@GFauxPas I've now completed the proof. Can you tidy it?
Given a real function $f$, if $f(x)$ is differentiable everywhere, then is the following true?
$f'(a)>0 \implies \exists b \in \Bbb R: f(b) > f(a)$
 
Let E/F an extension and T a subset of E, that spans E algebraically over F.
We have that $T=B\cup \{b_1, \ldots , b_m\}$. Does it holds that B spans also E algebraically over F ?
 
@MaryStar können Sie mir helfen?
 
So formal :O
 
well I don't know which environments are considered formal
I've seen her exercises use "Sie"
@SteamyRoot Kannst du mir helfen?
 
Written text is (nearly) always for formal. For a chatroom conversation, "du" should be fine :)
 
4:10 PM
I see, thanks
 
@DHMO Assume there ain't. Then $a$ is a maximum. That forces $f'(a) = 0$, no?
 
@DHMO Ich werde ein bisschen später Zeit haben :-)
 
@BalarkaSen sure but I don't know which theorem to use
 
Hmmm
 
Fermat's theorem.
 
4:12 PM
What if the derivative isn't continuous.... hmmm
 
Doesn't really matter, @Steamy
 
@DHMO you know I did the hard part
 
@BalarkaSen but I'm thinking of something like mean value theorem
 
Hrmm...
 
@GFauxPas as if my part weren't hard
 
4:13 PM
So you're literally asking for a proof of Fermat's theorem? There are plenty on google.
 
@BalarkaSen no i'm not...
 
I'm joking :D
 
Then what are you asking? I don't even understand the question
 
if you prove the hypothesis is false you dont need to prove antyhing about the conclusion so you can make the proof a little shorter
 
$$f(x) = \left\{\begin{array}{ll}
x & \text{ if } x \leq 0\\
x-3 & \text{ if } 0 < x < 2\\
... & \text { if } x \geq 2
\end{array} \right.$$
 
4:16 PM
Unless you want to get rid of the split between forward and reverse implication and prove iff the whole way through
 
@SteamyRoot it isn't differentiable when x=0
 
It's not even continuous there
 
@GFauxPas I don't understand "the hypothesis is false"
 
Hmmm, yeah, right, I'm confusing things
 
@BalarkaSen I was just expecting mean value theorem
 
4:17 PM
$\forall x : P(x). \implies. Q(x)$
if you disprove the LHS it doesnt matter what the RHS is
 
If you're not continuously differentiable what you can say at best is that it might not be strictly increasing in a neighborhood of $a$, @Steamy. But you can still say it's not a extremum.
 
Yeah, that's what I was thinking...
 
Stuff like $x^2\sin(1/x) + x$
 
@GFauxPas so you're asking me to disprove "a=e"? I don't understand.
 
"Thus the right hand side is not true." "Thus the left hand side is not true."
 
4:19 PM
Hi chat
 
either we merge the necessary and sufficient conditions into one big proof, or we dont need to prove LHS is not true
 
@GFauxPas but why merge if you already have the necessary condition?
I used the necessary condition many times in proving the sufficient condition lol
 
right, so them, you don't need to show "thus the left hand side is not true"
maybe I'm confused
 
oh ok
(referring to just edited version) is this what you mean?
 
I've gotten 6 texts from my bus service telling me changes in the schedule and 1 text from amazon letting me know my order was shipped and so I was disappointed 7 times it wasnt my gf texting me :(
 
4:21 PM
@Semiclassical Been reading a small booklet of sort on Bose-Einstein statistics, which I found gathering dust on my bookshelf. Really neat stuff, although I only superficially understand it.
 
right exactly DHMO
 
@GFauxPas I'm saying that the second case contains so many lemmata that you can separate some results
for example I proved that the equality case of e^x >= x+1 only happens when x=0
more importantly I proved that a^x-x-1 is convex whenever a>0
 
@BalarkaSen Neat
 
I am forgetting, but you're a stat-mech person, yep?
 
Occasionally
 
4:24 PM
I alternate between saying "lemmas" because "lemmata" I'm afraid sounds pedantic, and saying "lemmata" because it's fun to say
 
@BalarkaSen somehow I know that "a is a maximum" implies "f'(a) = 0" but I'm not convinced that its contrapositive is true lol
 
it's not true
also it's not true either
 
??????
 
hey guys
 
Contrapositive, not converse
 
4:26 PM
$f(x) = -|x|+2$
has a max at $0$
$g(x) = x^3$ for the converse not being true
oh wait
contrapositive
but anyway, $-|x| +2$, so be careful
 
f'(0) does not even make sense for that
 
when we have derivation rules like $(e^u)' = u'e^u$, what are the limitations ? because if $u$ is 1, then $(e^u)'=u'e^u=0$. u needs to be not constant, right ?
 
right Balarka, so I'm saying that you need to be careful when you say "$a$ is a maximum implies $f'(a) = 0$
 
oh, sure. he indicated $f$ is everywhere diff before
 
Oh I didn't see that XD
@PearlSek when using the chain rule, it's less confusing if you don't use prime notation
 
4:29 PM
@PearlSek well it is true if u=1 also? u is a constant, so (e^u)' must be 0
 
@Semiclassical What kind of a person you are most of the time? Condensed-matter something something?
 
Yeah.
 
or if you do, use $f'(x)$ because then you show what you're differentiating with respect to
 
i am stupid sorry
:facepalm:
 
You're not stupid, I've noticed a lot of people think they're stupid if they dont understand something immediately in math
don't know why that is, but the "stupid" people are the people who are afraid to ask and so never learn
so in other words, you're stupid not because you asked, but because you called yourself stupid. ?????
I'll be quiet now
anyway Pearlsek check out the Khan Academy videos on implicit differentiation
they helped me understand the chain rule when I was in calc I
 
4:32 PM
i try to resist calling myself stupid when I make a mistake. I prefer "silly"
Seems healthier
 
@GFauxPas are you following lol
 
@Semiclassical This is an impulse I could use.
 
My good friend Kevin Carlson posted his first (non-REU) paper today. I have no idea what it says.
 
@PearlSek Looks like 'thestudentroom.com' syntax
 
Sweet, I got an A in Physics II!!
 
4:45 PM
Congrats!
 
:)
ty
 
@MikeMiller Nice, but the paper looks scary.
 
@GFauxPas should we extract some paragraphs as lemmas/theorems?
 
Maybe DHMO but I don't feel like it righ tnow sorry :(
 
sure
 
4:46 PM
@BalarkaSen I'm trying to read it now.
 
Anyone seen Herbert Gross' MIT complex analysis lectures?
 
@MartinSleziak Oh! Okay. I thought maybe you were replying to something I said.
 
I wonder if I'll ever have to go back and learn non-analytic geometry
I never learned it
 
Like intrinsic geometry?
 
Euclid
compass and straightedge stuff
 
4:52 PM
Oh that stuff
I doubt it
 
I want to go into differential geometry, just because I've asked 3 professors what grad math I should go into , and I told them my favorite subjects, and they all said "differential geometry"
 
What are your favourite subjects, @GFauxPas?
 
I think most of the Euclidean geometry you'd need can be picked up from working through a bunch of geometric constructions. There's a couple dozen of them you'll ever need in your life.
 
I told them I like matrix calculus and smooth curves/surfaces/hypersurfaces
Find the solutions to $\mathbf J = \mathbf 0$ on $w = x^2 + \sin 2xyz$, things like that
well no, that's not really a good exmaple of what I like
 
@GFauxPas in light of the fact that characterisation is used throughout Proofwiki, could you move the page accordingly?
 
4:57 PM
Eyooo
 
basically I love Calculus and Linear Algebra
Both are used I think DHMO, we never had a consensus as to whether to use British or American English
one of my professors says that if you dig deep enough into any topic that's based on Calculus you'll find linear algebra, do you guys agree
 
They go pretty well together
I'd try reading some texts on functional analysis
 
Well, as soon as you come across a Jacobian you're in linear algebra, no?
 
ya agreed
 
@GFauxPas ok
 
5:03 PM
@DHMO it's that primemover is the biggest contributor, that's why
I think
 
@GFauxPas Is there anything I can do now?
 
you mean, for that proof?
 
for anything on ProofWiki
 
I think PW likes $\vdash\ \ \dashv$ \dashv \vdash instead of $\therefore \ \ \because$
um you can help me with my stuff leading up to laplace trasnforms :D
 
link?
 
5:08 PM
basically i'm not sure what the most elementary way is to deal with the origin in these
for a function to be of exponential order, it needs to be piecewise continuous at the origin from the right, and I know how to prove that these things are, but I don't know what the exposition should be
 
@TheGreatDuck That was a joke...
Sorry for the confusion I guess.
 
I really need to learn more LaTeX than mathjax
I'm going to have to write papers evnetually
 
@GFauxPas isn't it continuous at the origin also?
 
do I need to prove it, or is it sufficiently obvious?
Is $0^e = 0$ obvious?
it's not $\exp \left({e \ln 0}\right)$.
 
look, rational powers aren't defined using exp and ln
 
5:19 PM
so that takes care of that
 
yes
 
so what about $t^e \vert_{t = 0}$
 
rational or real?
 
$t$ is a positive real
$e = \exp 1$
well
non-negative real
you see the problem?
the thing is, in the context of integration, it's not important. but I need to show it's not important.
 
so you're talking about the first link
well I'm confused
what the hell are the domains
 
5:24 PM
@GFauxPas There's really not much of a jump
All you need to get by is Google and a bit of editing
 
@GFauxPas I suspect a circular proof here proofwiki.org/wiki/…
@GFauxPas nice
 
Induction proofs often "feel" circular, I'm not good and identifying circular induction proofs
 
I have raised the objection in the talk page
please participate
 
@hichris123 as i said many times over. I was not referring to your comment. I was referring to an earlier comment by a moderator telling me to stay out of chat and that if I caused any more trouble I would be banned. Sorry for the confusion. I figured you were joking.
 
5:47 PM
@TheGreatDuck you vote to close your own questions, why is that ?
 
$\Large \mathfrak {u \ U} \ \mathcal U \ \mathscr U \ u \ U \operatorname{U \ u} \ \texttt {U u} $ favorite?
for the heaviside step function
probably best to go with default $u$
 
@LeGrandDODOM I really dont know why. I think I was being too harsh towards myself about what is and is not allowed on the site. I legitimately believed I would be banned for asking those questions.
 
Hey everyone, quick question, given a set $X$ and a basis $\mathcal{B}$ for a topology on $X$, then the result that a collection $\mathcal{T}$ generated by $\mathcal{B}$, is a topology on $X$ should be classified as a Theorem or merely just checking a definition?
 
$\mathfrak {u \ U}$
^those two
(but completly unrelated to your question, i just find them nice)
 
thanks
the quality of being bounded is "boundedness"?
 
6:05 PM
> Why did the chicken cross the Möbius strip?

To get to the other ... -- no, wait ...
 
@GFauxPas I think you should use $K$ instead of $M$
 
done. you can do that to my proofs without asking me if you think its a good idea to change things
 
@GFauxPas can an upper bound be smaller than a lower bound?
 
in absolute value, why not
 
6:11 PM
right
 
though I had to ask myself that question twice when writing the proof :)
 
@GFauxPas don't use $L$, just use $\max (U,0)$
 
you do it
 
so you don't need a lower bound
but you would need to change the name of the theorem as well
bounded above function etc
 
but if it's unbounded below I don't know if its of exponential order
is it?
 
6:12 PM
why?
 
can someone explain to me why is the usage of axiom of choice in any kind of proof not particularly liked?
I mean people always try proving things without using it
 
$-e^{x^2}$?
 
what's with the hatred for that poor axiom?
 
Soumy, some people don't like the AOC because it's equivalent to some counterintuitive things
 
@GFauxPas why isn't it order 0?
 
6:13 PM
@GFauxPas for example?
 
Banach-Tarski Paradox
Because $\vert e^{x^2} \vert > e^x$?
 
@GFauxPas oh, sorry, my fault
 
Except AOC is also equivalent to plenty of things that are very intuitive
Like that every vector space has a basis
 
@GFauxPas would it be a good idea to replace "K > ..." with "K = ... + 1"?
 
and that the cartesian product of nonempty sets is nonempty
 
6:15 PM
@GFauxPas it is not intuitive that R/Q has a basis
 
hm
has a basis in $\mathbb Q$ you mean?
 
An old joke: "The Axiom of Choice is obviously true, the well-ordering principle obviously false, and who can tell about Zorn's lemma?"
3
 
has a basis over what?
 
@Semiclassical why would the well ordering principle be false?
 
i've no idea what it means, i just repeat it lol @GFauxPas
 
6:17 PM
Soumy those things are all equivalent to the axiom of choice
:)
 
@GFauxPas damn is there absolutely no proof of basis of vector space without AOC? :O
 
It's a joke. The axiom of choice, the well-ordering principle, and Zorn's lemma are all equivalent.
 
oh
 
@SoumyoB it just means that the principle is counter-intuitive
 
the well ordering principle?
 
6:18 PM
AFAIK Soumy in ZF it's equivalent
 
@SoumyoB yes
 
I don't even understand all the forms of the Axiom of Choice
 
Hello everyone
I just learned about iterated function systems, fractals and the chaos game.

Is the following statement correct: The chaos game uses iterated function systms to render fractals. ?
 
I should note that the above joke is credited to Jerry Bona.
 
the chaos game ?
 
6:21 PM
Still one of the best jokes if you've dealt with all 3 a bit :D
 
never mind actually I found the answer to my question just on wiki
Thanks :)
 
@GFauxPas so you're studying order now?
 
In mathematics, the term chaos game originally referred to a method of creating a fractal, using a polygon and an initial point selected at random inside it. The fractal is created by iteratively creating a sequence of points, starting with the initial random point, in which each point in the sequence is a given fraction of the distance between the previous point and one of the vertices of the polygon; the vertex is chosen at random in each iteration. Repeating this iterative process a large number of times, selecting the vertex at random on each iteration, and throwing out the first few points...
 
i'm studying laplace transforms
every function of exponential order has a laplace transform
sufficient, not necessary
and probably other integral transforms as well
 
@mercio Howver I don't see the link with chaos theory...
 
6:24 PM
@GFauxPas magic: proofwiki.org/w/…
 
-__-
 
do you like it?
 
maybe cosine and sine are important enough to have their own proofs
I mean the proofs are so short already
I dunno
 
I don't see the difference
 
hmm you're right
 
6:25 PM
If you like it then I'll perform the same magic here
 
No keep that one
you can change the sine one though
 
done
 
thanks
constant is so much "more" than bounded
so i think it should have its own
 
lol
@GFauxPas can I change the constant though
I found a nicer proof
 
what is it?
 
6:28 PM
well, |C| < (|C|+1)e^(0t)
no need that case splitting
 
hmm I think pm is changing your edits back
good call
 
nice! another meaningless bureaucratic arm-wrestling
i'll split proofs then
 
actually, @Semiclassical, I just noticed your joke is wrong :O
 
hey just want to check if my notation is correct.
 
It's the "well-ordering theorem", not the principle (which is obviously true)
 
6:31 PM
@GFauxPas what's happening between link1 and link2?
 
That might explain @SoumyoB 's confusion about why it would be false.
 
Because $\mathcal L \{ \sqrt t \}$ is simpler than other rational powers
and I'll use it later
 
@GFauxPas also, I'm not seeing $\forall t \ge M$ in any of your proofs yet it is stipulated clearly in the definition
 
also you can define square roots before defining general rational powers
 
@GFauxPas is there any Laplace transform that links to those results?
 
6:33 PM
I haven't gotten to them yet
Sometimes I say "for $t$ sufficiently large", sometimes I forget
Like proofwiki.org/wiki/… I say $t > 1$
 
then fix them
 
ugh DAD
later, I've been dealing with proofwiki for long enough today
 
Very handwavey
but I need to go now
 
how do you want me to fix it
 
I'll fix it when I come back
 
6:36 PM
k
 
I think Zorn's lemma actually feels intuitively true if one spends 10 minute understanding the definition of the objects involved and what the lemma is saying
 
Hmmm... I don't know.
 
what counter intuitive results besides Banach Tarski are equivalent?
I know that's the big one
 
It feels natural for sets with certain structure order than being a poset
The Well-Ordering Theorem is pretty counter-intuitive (hence the quote)
 
I mean it's not like it's obvious, but it's reasonable at least
 
6:45 PM
@SteamyRoot that may be true, but it's not the quote
 
@SteamyRoot I have to agree that seems more reasonable
 
I guess it is pretty weird that every set can be "put in order"
 
@Alessandro I think of it in terms of some of it's most obvious algebraic applications.
 
@Alessandro True, but in my opinion it's reasonable as well that for certain things it would not work.
 
Maximal ideals? @balarka
 
6:47 PM
Also that f.g. implies Noetherian.
 
@GFauxPas Exactly. For rather easy sets it can already be incredibly difficult to come up with a well-ordering.
Let alone any set.
 
I should reparse. What I mean is the fact that Noetherian (ascending chain condition) implies every nonempty collection of submodules has a maximal element.
 
Hm, but is it true that f.g. implies Noetherian?
 
@trilolil I don't think there is a link
 
f.g. is equivalent to being Noetherian, in fact
 
6:50 PM
@Semiclassical True, but it ruins the joke. Annoyingly, the joke is first cited in a 1997 handbook referring to the source as "private communication in 1977". So there's no telling if Bona misspoke, or if the first person writing down the joke wrote down the wrong thing. :P
 
Every ideal being f.g. is equivalent to Noetherian? Or is it enough for the ring?
 
The former.
I am being sloppy today, sorry.
 
No problem, but that looked suspicious :P
 
Hi @Alessandro, sloppy @Balarka
 
Morning, @Ted
 
6:53 PM
Hi @Ted
 
(I guess it's about midday there)
 
A bit before ...
 
Am watching a video on complex analysis and supposedly the Cauchy Riemann equations lead us to those differentials being exact. I'm not sure how it follows :-/
 
Not true. They're closed, hence locally exact.
Or exact on a star-shaped domain.
 
The point being locally a holomorphic function has an antiderivative. Globally that's false for many reasons (can you come up with an example of one?)
 
6:58 PM
Many reasons? ...One reason.
 
Ted are you familiar with the CUNY system
 
not hugely
 
I just read about star regions. Seems ok. You just gotta be able to find a point from which every other point is reachable by a line segment entirely contained within the shape.
 
I need to take some courses to shore up my math education to get ready for a pure math phd program
 
No idea what closed differentials are :-/
 
6:59 PM
my major is in applied math
 

« first day (2332 days earlier)      last day (2694 days later) »